7
$\begingroup$

I'm have some difficulties in bounding the following inequality:

I want to find a c as small as possible s.t. $$\sum_{i=1}^nx_i^4\sum_{i=1}^nx_i^2 -\sum_{i=1}^nx_i^6 \leq c\left(\sum_{i=1}^nx_i^3\right)^2$$

where $x_i$ are all non-negative

I know from the cauchy-inequality that

$$\sum_{i=1}^nx_i^4\sum_{i=1}^nx_i^2 \geq \left(\sum_{i=1}^nx_i^3\right)^2$$

But I think it useless in my question..

And more generally for some k and l, find out a small c s.t. $$\sum_{i=1}^nx_i^{2k-l}\sum_{i=1}^nx_i^l -\sum_{i=1}^nx_i^{2k} \leq c\left(\sum_{i=1}^nx_i^k\right)^2$$

Anyone help with out? Thanks!!

$\endgroup$
5
  • $\begingroup$ Try setting all $x_i$ equal to $1$. $\endgroup$
    – Yemon Choi
    Jun 1, 2012 at 7:12
  • $\begingroup$ Yemon, are you sure? I would expect the answer here not to be in closed form for n large enough. $\endgroup$ Jun 1, 2012 at 7:50
  • 1
    $\begingroup$ Gjergji, I understood the question as asking for the smallest $c$ independent of $n$ $\endgroup$
    – Yemon Choi
    Jun 1, 2012 at 8:07
  • $\begingroup$ Compute the case where the numbers are in geometric progression? $\endgroup$ Jun 1, 2012 at 8:35
  • $\begingroup$ We can show that for all natural $n\leq9$ we have $c=1-\frac{1}{n}.$ $\endgroup$ Jan 31, 2019 at 8:23

3 Answers 3

9
$\begingroup$

I will try to give an estimate. Represent your inequality as: $$ \sum_{i\lt k} (x_i^4x_k^2+x_i^2x_k^4) \le c(\sum_i x_i^6 + \sum_{i\lt k} x_i^3x_k^3). $$ There are $n(n-1)/2$ pairs of $i\lt k$, and every $i$ comes in $n-1$ pairs. Distributing this into pairs, we have: $$ \sum_{i\lt k} \frac{c}{n-1}(x_i^6 +x_k^6) + 2cx_i^3x_k^3 - x_i^2x_k^4-x_k^2x_i^4 \ge0. $$ Denote $a=(n-1)/c$ and consider one single pair with $x_i=x$, $x_k=y$: $$ x^6 + y^6 + 2(n-1)x^3y^3 - ax^2y^4-ax^4y^2\ge0. $$ All monomials are uniform (or what is the term?), so we can assume that $y=1$: $$ x^6 + 2(n-1)x^3 - ax^2-ax^4 +1 = (x^2+1) (x^4-(a+1)x^2+1) +2(n-1)x^3\ge0. $$ The biquadratic polynomial $(x^4-(a+1)x^2+1)$ has minimum at $x_0^2=(a+1)/2$, and this minimum equals $1-(a+1)^2/4 = 1-x_0^4$. If $x_0\le1$, i.e. $a=1$, then this is nonnegative and the whole expression is nonnegative. Thus, we already have an estimate: $a_{max}\ge 1$, $c_{min}\le n-1$.

Take now the term with $x^3$ into consideration. Still at the minimum point $x_0$, we have: $$ (x_0^2+1)(1-x_0^4)+2(n-1)x_0^3\ge0. $$ Of course we are interested in $x_0\ge1$ and $n\ge3$. One estimate I can guess is to put $2(n-1)=\alpha x_0^3$, then we want that: $$ (\alpha-1)x_0^6-x_0^4+x_0^2+1\le0, $$ what is of course true for all $x_0\ge1$ if $\alpha=1$, i.e. $x_0=(2(n-1))^{1/3}$. This gives an estimate on $c$ as something like $2^{-5/3}n^{1/3}$... By my methods one scarcely gets much better.

$\endgroup$
2
  • 4
    $\begingroup$ $n^{1/3}$ is the right order of magnitude: take $1,n^{-1/3},\dots,n^{-1/3}$. One can fight for the exact constant, of course, but I doubt it makes any difference to the OP whether it is $2^{-5/3}n^{1/3}$ as in Yulia's proof, or $2^{-2}n^{1/3}$ as in my simple-minded example. $\endgroup$
    – fedja
    Jun 1, 2012 at 12:10
  • $\begingroup$ Thanks for your help, this is really a good approach!! If it's true I think I have to try another way to attack my problem :( $\endgroup$
    – xwangae
    Jun 2, 2012 at 1:59
1
$\begingroup$

You can try Ozeki’s inequality, which is one of a number of known "reverse Cauchy-Schwarz inequalities" and seems best adapted to your situation. Have a look here:

http://www.ajmaa.org/RGMIA/papers/v6n4/RCBSInTCN.pdf

$\endgroup$
4
  • $\begingroup$ These Kantorovich-type inequalities are sort of a different flavor, and arise when you have control over the upper and lower bounds on the variables. $\endgroup$ Jun 1, 2012 at 10:57
  • $\begingroup$ Well, we don't know (yet) the context in which the problem arised. Maybe the OP has such control. I hope he will tell us. $\endgroup$ Jun 1, 2012 at 11:38
  • $\begingroup$ Thanks for your help, here $x_i$ is the frequency of number $i$, so it ranges from 0 to m, where m is given. I'm reading it but some of them requires $x_i \neq 0$ and this is not satisfied in this context. $\endgroup$
    – xwangae
    Jun 2, 2012 at 1:54
  • $\begingroup$ Then you can apply the inequalities to $y_{i}=x_{i}+1$. $\endgroup$ Jun 4, 2012 at 21:12
1
$\begingroup$

Let $x_i=1$ for all $i$.

Thus, $c\geq1-\frac{1}{n}.$

We'll prove that our inequality is true for $c=1-\frac{1}{n}$ for all natural $n\leq9.$

Indeed, we need to prove that $$\frac{n(n-1)}{n!}\sum_{sym}x_1^4x_2^2\leq\left(1-\frac{1}{n}\right)\left(\frac{n}{n!}\sum_{sym}x_1^6+2\cdot\frac{\frac{n(n-1)}{2}}{n!}\sum_{sym}x_1^3x_2^3\right)$$ or $$\sum_{sym}(x_1^6+(n-1)x_1^3x_2^3-nx_1^4x_2^2)\geq0$$ or $$\sum_{sym}(x_1^6-nx_1^4x_2^2+2(n-1)x_1^3x_2^3-nx_1^2x_2^4+x_2^6)\geq0.$$ Now, let $x_1=tx_2.$

Thus, it's enough to prove that $$t^6-nt^4+2(n-1)t^3-nt^2+1\geq0$$ or $$(t^3-1)^2\geq nt^2(t-1)^2,$$ for which it's enough to prove that $$(t^2+t+1)^2\geq9t^2,$$ which is true by AM-GM.

I used $\sum\limits_{sym}$ in the following sense.

For example for four variables $a$, $b$, $c$ and $d$.

We know that $|S_4|=4!=24$.
$a+b+c+d$ it's a sum of $4$ addends, which says $$a+b+c+d=\frac{4}{4!}\sum_{sym}a=\frac{1}{6}\sum_{sym}a$$ or $$\sum_{sym}a=6(a+b+c+d).$$ Similarly, $$ab+ac+bc+ad+bd+cd=\frac{1}{4}\sum_{sym}ab$$ or $$\sum_{sym}ab=4(ab+ac+bc+ad+bd+cd).$$ Also, we have for example $$\sum_{cyc}a^2(b+c+d)=\frac{1}{2}\sum_{sym}a^2b$$ by the same reasoning.

$\endgroup$
2
  • 1
    $\begingroup$ I believe you consider the notation $\sum_{sym}$ standard, by many of us will have a tough time trying to guess what it means, so you'd better have it explained. $\endgroup$
    – Seva
    Jan 31, 2019 at 9:06
  • 1
    $\begingroup$ @Seva I added something. See now. Wow!!! Seva, you don't remember me? $\endgroup$ Jan 31, 2019 at 9:22

Your Answer

By clicking “Post Your Answer”, you agree to our terms of service and acknowledge you have read our privacy policy.

Not the answer you're looking for? Browse other questions tagged or ask your own question.